王进明初等数论习题详细解答2013.5第九版(可打印版)

更新时间:2024-06-08 14:45:01 阅读量: 综合文库 文档下载

说明:文章内容仅供预览,部分内容可能不全。下载后的文档,内容与下面显示的完全一致。下载之前请确认下面内容是否您想要的,是否完整无缺。

王进明 初等数论 习题及作业解答

P17 习题1-1 1,2(2)(3), 3,7,11,12为作业。

1.已知两整数相除,得商12,余数26,又知被除数、除数、商及余数之和为454.求被除数.

解:a?12b?26,a?b?12?26?454,12b?26?b?12?26?454,

(12?1)b?454?12?26?26?390,b=30, 被除数a=12b+26=360.

这题的后面部分是小学数学的典型问题之一——“和倍” 问题: 商为12,表明被除数减去余数后是除数的12倍,被除数减去余数后与除数相加的和是除数的(12+1)倍,即454?12?26?26?390是除数的13倍.

2.证明:(1) 当n∈Z且n?9q?r(0?r?9)时,r只可能是0,1,8; 证:把n按被9除的余数分类,即:若n=3k, k∈Z,则n?27k, r=0; 若n=3k +1, k∈Z,则n?(3k)?3(3k)?3(3k)?1?9k(3k?3k?1)?1,r=1; 若n=3k-1, k∈Z,则n?(3k)?3(3k)?3(3k)?1?9(3k?3k?k?1)?8,r=8.

332323322333n3n2n??的值是整数。 (2) 当 n∈Z时,

326n3n2n2n3?3n2?n32??=证 因为,只需证明分子2n?3n?n是6的倍数。 32662n3?3n2?n?n(2n2?3n?1)?(n?1)n(2n?1)

?(n?1)n(n?2?n?1)=n(n?1)(n?2)?(n?1)n(n?1).

由k! 必整除k个连续整数知:6 |n(n?1)(n?2),6 |(n?1)n(n?1). 或证:2!|(n?1)n, (n?1)n必为偶数.故只需证3|(n?1)n(2n?1).

若3|n, 显然3|(n?1)n(2n?1);若n为3k +1, k∈Z,则n-1是3的倍数,得知

(n?1)n(2n?1)为3的倍数;若n为3k-1, k∈Z,则2n-1=2(3k-1)-1=6k-3, 2n-1

是3的倍数.

综上所述,(n?1)n(2n?1)必是6的倍数,故命题得证。

(n?1)n(2n?1)222

=0+1+2+?+(n-1)2,整数的平方和必为整数。

6(n?1)n(2n?1)-

当 n∈Z时,-n∈Z+, 从而同样推得为整数,故命题得证。

6又证:

(3) 若n为非负整数,则133|(11n+2+122n+1).

1

证明:利用11n+2+122n+1=121×11n +12×144 n =133×11n +12×(144 n-11 n)及例5的结论. (4)当m,n,l∈N+时,

(m?n?l)!的值总是整数

m!n!l!证明:(m?n?l)!=(m?n?l)(m?n?l?1)?(n?l?1)(n?l)(n?l?1)?(l?1)?l! 由k!必整除k个连续整数知:m!|(m?n?l)(m?n?l?1)?(n?l?1), n! |(n?l)(n?l?1)?(l?1),从而由和的整除性即证得命题。 (5)当a,b∈Z且a ≠-b,n是双数时,?a?b?|(an?bn); (6)当a,b∈Z且a ≠-b,n是单数时,?a?b?|(a?b).

nn解:利用例5结论:若a ≠ b,则?a?b?|(a?b).令b=-b*, 即得。

nn或解: a = (a+b)-b, (5) 当n为双数时,由二项式展开

nan?bn??a?b?b?b? ????n??a?b??n?a?b?nn?1b?????1?n?1n?a?b?bn?1,证得。(6) 当n为单数时类似可得。

3.已知a1,a2,a3,a4,a5,b∈Z,且

?ai?152i?b2,说明这六个数不能都是奇数.

解:若这六个数都是奇数,设ai?2ki?1,ki?Z,i?1,2,3,4,5,则

?a??(2k?1)2iii?1i?1552?4?ki(ki?1)?5,因为2|ki(ki?1),所以8 | 4?ki(ki?1),

i?155i?1?ai?152i?8q?5,q?Z, 而b2?(2k?1)2?4k(k?1)?1,b2?8q*?1,k,q*?Z,

即等式左边被8除余5, 而右边被8除余1, 故不可能这六个数都是奇数。

4.能否在下式的各□内填入加号或减号,使下式成立;能的话给出一种填法,否则,说明理由。

1□2□3□4□5□6□7□8□9=10

不能,因为等式左边有单数个单数,它们的和差只能是奇数,而等式右边10为偶数。或解:无论各□内填入加号或减号,1□2□3□4□5□6□7□8□9+1+2+3+4+5+6+7+8+9总是偶数,而1+2+3+4+5+6+7+8+9=45,因此的结果1□2□3□4□5□6□7□8□9一定是奇数。 5.已知:a,b,c均为奇数.证明ax?bx?c?0无有理根。 证:若有有理根,记为

2ppp,p,q互质,代入方程有a()2?b??c?0

qqq 2

即ap2?bpq?cq2?0,这是不可能的,因为p,q互质,二者不可能同时为偶数。 若p为偶数,则ap2?bpq为偶数,但cq2是奇数,它们的和不可能为0; 若q为偶数,则bpq?cq2为偶数,但ap2是奇数,它们的和也不可能为0。

6.在黑板上写出三个整数,然后擦去一个,换成其他两数之和加1,继续这样操作下去,最后得到三个数为35,47,83.问原来所写的三个数能否是2,4,6?

解:不能.因为原来所写的三个数若是2,4,6,每次操作后剩下的三个数是两偶一奇.

7.将1-—99这99个自然数依次写成一排,得一多位数A=1 2 3 4 5 6 7 8 9 1011…97 98 99,求A除以2或5、4或25、8或125、3或9、11的余数分别是多少?

解:由数的整除特征,2和5 看末位,∴ A除以2余1,A除以5余4;4和25 看末两位,∴ A除以4余3,A除以25余24;8和125看末三位,∴ A除以8余3,且除以125余24;3和9看各位数字的和,1+2+3+4+5+6+7+8+9=45,A所有数字的和等于450, ∴ A除以3和9都余0,A除以11的余数利用定理1. 4, 计算奇数位数字之和-A 的偶数位数字之和.奇数位数字之和1+3+5+7+9+(0+1+…+9) ×9,偶数位数字之和2+4+6+8+(1+2+…+9) ×10,两者之差为-40,原数除以11的余数就是-40除以11的余数:4.

8.四位数7x2y能同时被2,3,5整除,求这样的四位数.

解:同时被2,5整除,个位为0,再考虑被3整除,有4个:7020,7320,7620,7920. 9.从5, 6, 7, 8, 9这五个数字中选出四个不同的数字组成一个四位数,它能同时被3, 5, 7整除,那么这些四位数中最大的一个是多少?

被5整除,个位必为5. 5+6+7+8=26, 5+6+7+9=27 ,5+6+8+9=28,5+7+8+9=29中唯27能被3整除,故选出的四个不同的数字是5, 6, 7,9,但不同排序有9765,9675,7965,7695,6975,6795,

从最大的开始试除,得9765=7×1395,那么要求的就是9765了。 10.

11.1至1001各数按以下的格式排列成表,像表中所示的那样用—个正方形框住其中的9个数,要使9个数的和等于(1)2001,(2)2529,(3)1989,能否办到?如能办到,写出框里的最小数与最大数.如办不到,说明理由.

181522291623310172441118255121926613202771421 28?????99599699799899910001001解:设框里居中心的数为x,则9个数的和等于9x. (1) 9不能整除2001,∴和等于2001办不到;(2) 9x=2529,x=281,是所在行第一个数,∴和等于2529办不到;(3) 9x=1989,x=221,和等于1989能办到,框里的最大数为x+8=229,最小数为x-8=213.

12.证明:7(或11或13) |anan?1?a3a2a1a0的特征是:7(或11或13) 整除

|anan?1?a3?a2aa| 10解答:因为7×11×13=1001。(谐“一千零一夜”)而

3

anan?1?a3a2a1a0=7×11×13×a2a1a0?(anan?1?a3?a2a1a0)×1000.

或 anan?1?a3a2a1a0?anan?1?a3?7?11?13?(anan?1?a3?a2a1a0)

附)广西师范大学赵继源主编的《初等数论》习题1—1中的部分题目

(与上面相同或相似的题目不列,以下各章节同)

3.已知a,b,c中,有一个是2001,有一个是2002,有一个是2003,试判断(a—1)×(b—2)×(c—3)的奇偶性,并说明理由. 6.24|62742??,求?,?.

9. 是否存在自然数a和b,使a2-b2 = 2002成立? 11.证明:当n∈Z时,6 | n(n+1)(2n+1).

12.已知:f?x??ax?bx?c,f (0),f (-1),f (1),x均为整数.证明:f?x??Z.

2 解答:

3.偶数.因为a,b,c中,有三个奇数,所以a-1,c-3中至少有一个是偶数. 6.只需3|62742??,且8|62742??,即3|(???),且8|??,先考虑??0,2,4,6,8,有5组解 ????0,???2,???4,???7,???9, ???????0;???4;???8;???2;???6.9.不存在.利用a2-b2 =(a-b)(a + b),而a-b,a + b的奇偶性相同.而2002=2×1001. 11.用数学归纳法或n(n+1)(2n+1)= n(n+1)(n+2)+(n-1)n(n+1),利用整除的基本性质(13). 12.由f (0),f (-1),f (1),x均为整数可得c, a+b, a-b均为整数. 进而知2a,2b为整数.

分类讨论(k∈Z): x=2k时,由2a,2b为整数f (x)显然为整数;

x=2k+1时,f (2k+1) = 4ak(k+1) + 2bk + a + b + c, 可知f (x)仍然为整数。

习题1-2

1. 用试除法确定下列各数中哪些是质数?哪些是合数?1987,2027,2461,17357 解:1987?45,

2027≈45.022, 用质数试除到43,可知两者是质数, 2461≈49.61,

用质数顺次试除,试除时,用数的整除特征考虑:2,3,5显然不能整除它,由上节第12题结论,357-17= 340,340不能被7,11,13整除,再用17,23考虑,2461÷23=107, 2461是合数。用类似思路顺次试除17357,到17,得分解式17357=17×1021,17357是合数。 2. 当 n 是什么正整数时,f1(n)?n?4,f2(n)?n?5n?9n+8n2+4n +1, f3(n)= n4

4543-18n2+45, f4(n)= n4+ n2+1, f5(n)?3n?4n?1的值是质数?是合数?

2解:f1(n)?n?4n?4?4n?(n?2)?(2n)?(n?2n?2)(n?2n?2)

42222222?[(n?1)2?1][(n?1)2?1],当n =1时,f1(n)是质数;当n >1时,f1(n)是合数。

4

f2(n)?(n5?2n4?n3)?(3n4?6n3?3n2)+(2n3?4n2?2n)?(n2?2n?1)

=( n +1)2(n3+3n2+2n +1)。n 无论是什么正整数时,n +1>1,∴f2(n)总是合数。

f3(n)=(n2?3)(n2?15)令n2-3=1或n2-15=1知仅当 n=2或 n= 4时,f3(n)为质数,n为其它正整数时,f3(n)是合数。

∵ n4+ n2+1 = n4+ 2n2+1-n2 = (n2+ n+ 1)(n2-n+ 1),令n2+ n+ 1=1或n2-n+ 1=1知 仅当 n= 1时,f4(n)=n4+ n2+1为质数, n >1时,f4(n)是合数。

f5(n)?(3n?1)(n?1),∴f5(n)当n=2时为质数,n为其它正整数时是合数。

3. 试证:(1) 一切大于3的质数,不是形如6n +1就是6n-1的数(n∈N);

(2) 任意多个形如6n +1的数的乘积仍是形如6n +1的数; (3) 形如 6n-1的数中含有无限多个质数.

证:(1)因为形如6n或6n±2或6n+3为合数;所以结论成立; (2)先证明两个形如6n +1的数的乘积仍是形如6n +1的数:

(6n1?1)(6n2?1)?36n1n2?6n1?6n2?1?6(6n1n2?n1?n2)?1,

显然,6n1n2?n1?n2?N,结论成立。然后用数学归纳法可得一般性结论。 (3) 若形如 6n-1的数中只有k个质数:p1, p2, …, pk。令N = 6 p1 p2 … pk-1,N为形如 6n-1的数,由假设N必为合数,且必有一个形如 6n-1的质因数p(否则就全是形如 6n + 1的数,由(2)中结论,乘积必为形如 6n+1的数,与N的形式不符), 因此p为 p1, p2, …, pk中的某一个,于是,p | 1, 矛盾。

4. 设m>1,当m |[(m?1)!?1]时,m必为质数.

证:若m为合数,m最小的大于1的约数必为质数,设为p, p是2,3, …,m-1中一个数。 显然p | (m-1)!且p | [(m-1)!+1],于是,p | 1, 矛盾。

5. 是否有1999 个连续的自然数, 它们之中恰好只有一个是质数?

证:显然存在1998个连续的自然数都是合数, 比如1999!+2, 1999!+3, …, 1999!+1999. 现在设a1,a2,?,a1998是任意1998个连续的合数,若比a1小的最大的质数是 p, 则p,p+1,p+2,,?,p+1998 就是题目所要求的1999 个连续的自然数。

这里的p可以如下确定:比a1小1的不是质数, 就考虑比a1小2的,还不是,就考虑小3的,?,直到是质数为止,这个质数即为p。

事实上,p后面的合数不少于1998个, 所以可从p后面选1998个都是合数。 附)广西师范大学 赵继源主编的《初等数论》习题1—2中的部分题目(与以上相同的不列) 1. 判断下列各数中哪些是质数?109,2003

2. 求证:对任意 n∈Z+,必有 n 个连续的自然数都是合数. 4. 求证:当 n∈Z+时,4n3+6n2+4n +1是合数.

5

令(a,b)?d,a?dt1,b?dt2,?t1,t2??1,?x,y使xt1?yt2?1,x(t1?t2)?(y?x)t2?1, ?(t1?t2,t2)?1.同理(t1?t2,t1)?1.?(t1?t2,t1t2)?(t1?t2,t1)?1(定理1.21),

?(dt1?dt2,dt1t2)?d(定理1.14).即,?a,b??(a?b,?a,b?). ( 33, 90 ) = 3, 所以 ( a, b ) = 3.

10. 因为AB、BC 的中点上都要植上一棵树,315÷2=157.5因此应考虑1400和1575的最大公约数175。最后答案:两树间的距离最多有17.5米 . 11. 2个 .

12. 设小明 x岁,则爷爷 7x岁,7x +h =6(x+h) , x=5h; 7x +k =5(x+k) , x=2k; 7x +i =4(x+i) , x=i; 7x +j =2(x+j) , 5x=j; 知小明年龄是2, 5的倍数。因此小明 10岁,爷爷 70岁.

习题 1-4

1.把下列各数分解质因数:2001,26840,111111,999 999 999 999 解: 2001= 3×23× 29, 26840= 5×11×23×61, 111 111= 3× 7×11×13× 37. 999 999 999 999=111 111×9 000 009=33× 7×11×13× 37×1 000 001,而 1 000 001=101×9901,∴999 999 999 999=33× 7×11×13× 37×101×9901。 2.用分解质因数法求:(1)(4712,4978,5890,6327);(2)[4712,4978,5890]. 解:4712=4×1178=23×589=23×19×31, 4978=2×2489=2×19×131 5890=10×589=2×5×31, 6327=9×703=32×19×37 ∴ (1)19, (2)3086360×9×37=1027757880.

3. 将 85,87,102,111,124,148,154,230,341,354,413,667分成两组(每组 6个数),怎么分才能使每组各数的乘积相等?

解:—组为:85,111,124,154,354,667;另一组为:87.102,148,230,341,413. 4.某校师生为贫困捐款1995元,这个学校共有教职工35人,14个教学班,各班学生人数相同,并且多于30人但又不超过45人,如果师生平均每人捐款的钱数都是整数元,那么平均每人捐款多少元? 解:1995=3×665=5×399=7×285=5×7×57=3×5×7×19, 每人捐款的钱数只能是1、3、5、15、19、21或57。57不可能,否则教师捐款就够了; 19,15和1元也不行,与学生数显然不符; 平均每人捐款5元的话,每班人数为(399-35)=364÷14=26, 仍少于30; 所以平均每人捐款3元。此时每班人数为(665-35)=630÷14=45符合假定人数范围。

5. 甲、乙两人各射五箭,每射一箭得到环数或者是“0”(脱靶),或者是不超过10的自然数。两人五箭所得环数的乘积都是1764,但甲的总环数比乙的少4环求甲、乙两人的总环数各是多少?

解:1764=4×441=4×212=22×32×72, 由441-1=440=2×220,220=11×10×2 6.证明:(1) (a,[b,c]) = [(a,b),(a,c)]

2?a,b,c?.此题1-3已证明,在此要求用分解质因数方法证明。 [a,b,c](2)?[a,b][b,c][c,a](a,b)(b,c)(c,a)2r

(采用教材p成分的说法,可使证明叙述起来简捷一些:设p为质数,a,r∈N+,如果p | a, 且 p

r+1r

| a,这时称 p为 a 的 p 成分,用 p(a) 表示a的p成分的幂指数,即p(a) =r. )

证明:设p(a) =l,p(b) = m,p(c) =n,且不妨设l?m?n。

(1)p (a,[b,c]) =min{ p(a),max{ p(b), p(c)} }= min{l, m}=m,

11

p [(a,b),(a,c)]= max{ min{ p(a), p(b) }, min{ p(a), p(c)}}= max{m,n}= mr.

由于p是任意质数,所以(a,[b,c]) = [(a,b),(a,c)]。

(2)∵p([a,b][b,c][c,a])?p[a,b]?p[b,c]?p[c,a]?l?m?l?2l?m

?[a,b,c]2?p[a,b,c]?2p[a,b,c]?2l,∴p????m。

?[a,b][b,c][c,a]?2∵p((a,b)(b,c)(c,a))?p?a,b??p(b,c)?p(c,a)?m?n?n?2n?m

2??a,b,c??2???m。 p[a,b,c]?2p[a,b,c]?2n ∴p??(a,b)(b,c)(c,a)???由于p是任意质数,所以题设等式成立。

7. 自然数555 555的约数中,最大的三位数是多少? 解:555 555=5×111 111= 3×5×7×11×13× 37. 约数最小的三个相乘已经是三位数,那么在

C63?6?5?4?20种取法中, 3×5×37=555显然太小,故3,5只能取一个:5×11×13

3?2=55×13=715,再调大如7×11×13=1001就超出了;3×7×37=777,调大如3×11× 37=1 111,已经超出,故777即为所求。

8.若 2836,4582,5164,6522 四个数被同一个自然数相除,所得余数相同,求除数和余数各是多少?

解:4582-2836=1746=2×873=2×9×97, 5164-4582=582=2×291=2×3×97, 6522-5164=1358=2×679=2×7×97, (1746,582,1358)=2×97 所以,除数为97时,余数为23;除数为194时,余数为120. 9. (1) 所有正约数之和=15的最小自然数是多少? (2) 所有正约数之积=64的最小自然数是多少?

(3) 有没有这样的自然数,其所有正的真约数之积等于它本身?

解 (1) 15=1×15=3×5,若15=1×15,a有唯一质因数,即a=p,

由公式??a?=15?p??p??1???p?1,?p(p??1???p?1)?14,p只能是2. 令自然数a=2k,此时??a??2若??a??15=3×5,即

k?1??1?15,得k=3,即a=8;

?i?12pi?i?1?1?3×5

pi?13?pi?i?pi?i?1???pi?1?pi(p?i?1???pi?1)?2,pi,?i无解。

故??a??15=3×5为不可能。因此所求最小自然数就是8. (2)a

??a??64?26,a??a??212或a??a??46,a??a??84a??a??642。

?(2)=2≠12, ?(4)=3≠6, ?(64)=7≠2, ?(8)=4, 因此所求最小自然数就是8.

?a?a (3) 自然数a的所有正约数之积=a??,由题设有a???a2,即??a??4=22,

12

故分解式或 ⑴ 只含1个质因数 p,此时 p 的指数为3, 即a?p3, 或 ⑵ 为2 个质数的乘积,即a?p1p2,这样的自然数,其所有正的真约数之积都等于它本身。 2

10. 若a,b,c∈N+, 且 a= bc,(b,c)=1,则b,c均为平方数。 证明:?(b,c)?1,令b??pi?1mni?i,c??qjj,有pi?qj(?i,j)

j?1m?已知a?bc?2?pi??qj,故a??pi2??qj2,?ii?1j?1n?jn?im?j?i2?N?,?j2?N?(?i,j).

i?1j?1得?i?2?i',?j?2?j',?i'?N?,?j'?N?(?i,j).

???2?'?'??b??pi2?i'???pi?i'?,c??qjj???qjj?, 结论成立。

i?1j?1?i?1??j?1?nnmm2211. 975×935×972×( )要使这个乘积的最后 4个数字都是 0,括号中最小应填什么自然数?

解:20.四个数分解质因数后一共应该有且只有4个2与4个5,需补充2个2与1个5。

12. (1)设[a,b]=72,且a≠b,那么a + b有多少种不同的值? (2)已知(a,b)=12, [a,c]= [b,c]=300,满足上述条件的自然数共有多少组(a=12, b=c=300与a= c=300, b=12算不同的两组)? 解:(1)72=23×32, 由充要条件a,b只含有2,3的质因数且不超过72, 见下表 a b a+b 32 17 23=8 2×32 26 22×32 44 32 33 23×3=24 2×32 42 22×32 60 1 73 23×32=72 32 81 2×32 90 22×32 108 由于a,b是对称的,所以一共10种不同的值。

(2)300=12×25, 因此a,b,c只含有2,3,5的质因数且不超过300, 见下表 a b c 12 12×52 12 12×5 12×52 12×52 12×52 12×5=60 12 同左 12×52=300 12 同左 一共5组。 13. 求:(1)?(180);(2)?(180);(3)?1 (180).

23?133?152?1???7?13?6=546 (3)1809.解:180=2×3×5, ∴(1)3×3×2=18. (2) 2?13?15?12

2

14.求出最小的正整数n,使其恰有144个正约数,并且其中有十个是连续的整数。 解:144=122=6×3×2×2×2, 表明n的标准分解式应含有五个不同质数,并应该从2开始取,且使2的次数为5,3的次数为2,其他皆为1,故所求即25×32×5×7×11=110880。 它不仅有10个甚至有12连续的约数:1,2,3,4,5,6,7,8,9,10,11,12.

附)广西师范大学赵继源主编的《初等数论》习题1—4中的部分题目(与以上相同的不列)

13

6.200以内仅有 10个正约数的自然数有几个?并一一求出 . 7.求:(1)?(180);(2)?(180);(3)?1 (180). 8.已知[A,B]=42,[B,C]=66,(A,C)=3,求 A,B,C .

9.一个自然数有 21个正约数,而另一个自然数有 10个正约数,这两个数的标准分解式中仅含有不大于 3的质因数,且这两个数的最大公约数是 18,求此两数是多少?

10.小明有一个三层书架,他的书的五分之一放在第一层,七分之几(这个几记不清了)放在第二层,而第三层有书 303本,问小明共有书多少本?

11.某班同学(50人左右)在王老师带领下去植树,学生恰好能分成人数相等的 3 组,如果老师与学生每人种树的棵数一样多,共种了884棵,那么每人种多少棵树? 12.少年宫游乐厅内悬挂着 200个彩色灯泡,这 200个灯泡按 1耀200编号,它们的亮暗规则是:第 1秒:全部灯泡变亮;第 2秒:凡编号为 2的倍数的灯泡由亮变暗;第 3秒:凡编号为 3的倍数的灯泡改变原来的亮暗状态,即亮的变暗,暗的变亮 .一般地,第 n 秒凡编号为 n 的倍数的灯泡改变原来的亮暗状态。这样继续下去,每 4分钟一个周期,问第 200 秒时,明亮的灯泡有多少个?

习题 1-4解答

6.有5个,10=2×5=1×10因此所求的数应该为ab或c后者即令c=2也已经超出200,因此分别令a=2.b=3; a=2.b=5; a=2.b=7; a=3,b=2; a=2.b=11; 得48,80,112,162,176. 8.因为B |?B,C? , B |?A,B?, 所以B是66,42的公约数,因而B是6的约数。又

49因为?B,C??66?2?3?11,?A,B??42?2?3?7,所以7|A,11|C,从而设

因为知?2??2?1,且?1?1。A?2?13?27,B?2?13?2,C?2?13?211, 由?A,C??3,若B不含2的话,由?B,C??66, ?A,B??42,A,C就必须同时含2, 与?A,C??3矛盾。

∴A?2?1?3?7,B?2?3?2,C?2?1?3?11,?1,?2,?1?1,0,而且?1与?1不能同时为1. 于是?2?1和0时,各有?1?1,?1?0;?1?0,?1?0;?1?0,?1?三种情况1,共得6组解,分别为:(自行写出)

9. 576和162 10.3535本。解:由题目可知小明的书的册数是35的倍数, 设为35k, 可列出方程28k-5xk=(28-5x)k=303=3×101知k=101.

11. 分解质因数:884=4×13×17=17×52=68×13,884的因数中有4, 13, 52都具有3k+1形式,只有52=符合50人左右的题设,因此学生51人。

12. 灯的一次“改变”对应着它的编号的一个因子. 要使灯仍旧亮着需要奇数次“改变”.什么样的数有奇数个因子呢? 由定理1.26公式⑴知只有完全平方数! 200以内的完全平方数只有14个。即为答案. 此题也可先考虑10个灯泡。用归纳得出“只有完全平方数”的结论。

习题1-6部分习题解答

14

1. 若集A=?x|x?[sin?],??R?,B=x|x?[tan?],????2?k?,求A∩B,A∪B.

?解:A={-1,1}, B=Z(全体整数集合),∴A∩B=A,A∪B=Z. 2. 设 ?1??1?2,试求??,????(1?7)??的值。 ????3?7??3?7??3?77?1?1??3?7?解:, 代入得10。 ?2?7?3,??????2,???2?????2?22?3?7??3?3333. 求??1?2?3??2000??的值。

解:显然,1?2?33??32000?1;设1?2?33??32000?2

3则1?2?33??32000?8,2?33??32000?7,即2?3??32000?343。

33333333??32000?341,继续下去有3?34??32000?3413,这显然是不可能的,因此

?3?333??1?2?3??2000??=1.

4. 证明:若(p, q)=1,则??????????q??q??证:对于1?k?q-1,均有q ?k p,即??p??2p??(q?1)p?(p?1)(q?1)。 ??q?2kp?kp??kp??Z,故????????1 q?q??q?于是??kp??(q?k)p??kp??kp?=??p????p?1 ???????q??q??q??q??p??2p??(q?1)p?q?11??kp??(q?k)p??(p?1)(q?1)∴????。 ????????????????2?q??q??q?k?12??q??q??5. 求?log21???log22???log23?????log21024?的值。 解:?log21???log22???log23?????log21024?=

=?log22???log22???log24?????log24???log28?????log2512???log21024?

????????????????24=1×2+2×2+3×2+…+9×2+10=1 × (22-2)+2×(23-22)+3×(24-23)+…+9×(210-29)+10

239

210?1=-2-2-2-…-2+9×2+10=?2?+9×210+10=7×210+12=7180.

2?12

3

9

10

6. 求使

101?102?103???999?1000为整数的最大自然数k的值。 k715

解:即求101?102?103???999?1000的标准分解式中7的幂指数。

101?102?103???999?1000=1000!÷100!所以7(1000!÷100!)= 7(1000!)- 7(100!) 所以k=??1000??1000??1000??100??100???2???3?????2?=142+20+2-14-2=148. ???7??7??7??7??7?7. 解方程:(1)3x?5?x??50?0

解:x??x??x?1故3?x??5?x??50?0,3?x??3?5?x??50?0,解得

205047?x?. 代入原方程得3x=20, ?x??,?x??,??x??6.38850?5y50?3x50?3x8x?50或解:由已知得?x??设其为y,则x?,0??x??x???1,

3555205050?5y5547505055. 解得?,?y?, y?14 ?x??x?. 即?383888882(2)原式化为?x??x(x??x?)?0,即?x???x?x?x?0,把x当作常数,由一元二次

22方程求根公式得 ?x??5?15?15?1相乘的积为整数,只能是x?。 x,与222?y?2?x??3?y?3?x?2??5,且不是一个整数,则x+ y在哪两个整数之间。

8. 设x,y满足下列方程组?解:y?2?x??3?3?x?2??5?3(?x????2?)?5?3(?x??2)?5?3?x??1,

可得[x]=4,∴4?x <5,y=2[x]+3=11 ∴15< x+ y<16.

9. 试证方程[x]+[2x]+[4x]+[8x]+[16x]+[32x]=12345无实数解. 证: 假设方程有实数解x=n+a,其中n∈Z, 0?a<1,于是,,

[x]=n,[2x]=2n+[2a],[4x]=4n+[4a],[8x]=8n+[8a],[16x]=16n+[16a],[32x]=32n+[32a]。 代入原方程化简、变形,得 [2a]+[4a]+[8a]+[16a]+[32a]=12345-63n。 由于0?a<1,因而0?[ka]?k-1

故0?12345-63n?1+3+7+15+31=57,即12288/63?n?12345/63 亦即195.04?n?195.95,与n∈Z矛盾,故原方程无实数解。

10. 1到120这120个正整数中所有质数的和是多少?

解:2+3+5+7+11+13+17+19+23+29+31+37+41+43+47+53+59+61+67+71+73+79+83+89+97+ 101+103+107+109+113=1593.

或解:应用逐步淘汰原则,所求的和=1到120这120个正整数的和-(120以内,下同)2的倍数的和-3的倍数的和-5的倍数的和-7的倍数的和-11的倍数的和+6的倍数的和+10的倍数的和+14的倍数的和+22的倍数的和+15的倍数的和+21的倍数的和+33的倍数的和+35的倍数的和+55的倍数的和+77-30的倍数的和-42的倍数的和-66-70-110-105= =60?121?2(1?2?????120?)?3(1?2???40)?5(1?2???24)? ??2? 16

?120??7(1?2????)?11(1?2???10)?6(1?2???20)?10(1?2???12)

?7???120??14(1?2????)?22(1?2???5)?15(1?2???8)?21(1?2???5)? ??14?+33(1+2+3)+105+165+77-30(1+2+3+4)-42-84-66-70-110-105

=7260-61×60-61×60-3×41×20-5×25×12-7×17×8-11×11×5+6×21×10

+10×13×6+14×9×4+22×3×5+15×9×4+21×3×5+33×6+105+165+77-300-42-84-66-70-110-105=1593. 11. 求25!的标准分解式。 解:25!=222×310×56×73×112×13×17×19×23. 由定理1.6.3公式求出各个质数的指数。 12. 2000! 末尾有多少个连续的零? 解:??2000??2000??2000??2000?????400?80?16?3?499个连续的零

?5????52????53????54??13.某班学生参加数、理、化三科测试。数、理、化成绩优秀的学生人数依次是30,28,25, 数理、理化、化数两科成绩都优秀的学生人数依为20,16,17。数理化三科都优秀的学生有10人。问:数理两科至少有一科优秀的学生有多少人,数理化三科至少有一科优秀的学生又有多少人。

解:数理两科至少有一科优秀的学生人数为30+28-20=38;

数理化三科至少有一科优秀的学生30+28+25-20-16-17+10=40

14. 从自然数列1,2,3,4,?中依次划去3的倍数和4的倍数,但是其中凡是5的倍数的数均保留,划完之后的数依次构成一个新的数列:a1=1,a2=2,a3=5,a4=7,求a2000的值。 解:考虑1到3000这3000个数中这个数列中的数的个数为

?3000??3000??3000??3000??3000??3000?3000??????? ???4??3????3?4????3?5????4?5????3?4?5??=3000-1000-750+250+200+150-50=1800

?300??300??300??300??300??300?300????????????=180 ???????3??4??3?4??3?5??4?5??3?4?5??30??30??30??30??30??30?30???????????=30-10-7+2+2+1=18

?3??4??3?4????3?5????4?5????3?4?5??可知3330= a1998, 而3332是4的倍数,3333是3的倍数,所以a2000=3334.

附)广西师范大学赵继源主编的《初等数论》习题1—6中的部分题目(与以上相同的不列) 3. 若x,y?R?,?1?求证:?xy???x??y?; ?2?试讨论?xy?与?x??y?的大小关系。?证:?1??xy?????x??y?+?x??y?+?x??y??? ???x?+?x????y?+?y?????x??y??? 显然 ???x??y?+?x??y?+?x??y????0,证得。 17

?2?x,y?Z+时,?x???y???xy??0,有?x??y???xy?;而 x?1.2,y?2.1时,xy?2.52,?xy??0.52,?x??y??0.02,?x??y???xy?; 又 x?1.9,y?1.8时,xy?3.42,?xy??0.42,?x??y??0.72,?x??y???xy?.可见,三种情况都有。 7.计算:(1)??199?1??199?2??199?96? ?????????979797??????5?1??5?2?5?96??解: 原式??2?1? ?2?2????2?96???????979797???????5?1??5?2??5?96??2?1?2?2???2?96????????97????97???97??

?5?1??5?19??5?20??5?38??5?39??5?58??9312?????????????????97???97????97???97????97???97???5?78??5?96??????????97???97??=9312+0+…+0+19+40+57+76=9504.

(2)?1???111? ???223220082??111111????1???? 2232200821?22?32007?2008解: 考虑1??1?1?1111111??????2??2。从而=1. 1????222??2232200822008232008??8. 在前2001个自然数中,既不是5 的倍数,又不是7的倍数的数有多少个?

??. 2001??=2001-400-285+57=1373个

?5????7????7?5??10. 有100盏亮着的灯,各有一个拉线开关控制着,,现将其按顺序编上号码1,2, ?,100,然

后将编号为2的倍数的灯线拉一下,再将编号为3的倍数的灯线拉一下,最后将编号为5 的倍数的灯线拉一下,3次拉完之后还有几盏灯是亮的?49盏 11. 解方程:x?1?x?1??x?

?2001??2001??2001?x??1??2?解:已知x?1?x?1??2x?1?x?1, x??1时,?x???2,??2?x??1; ?2x?1??1?x?1时,?x??2x,即??x??0??1?x?0,即?2?2x?0,而2x?Z,x?1时,?x??2,?2?x?3.?x??,x?0;

∴ -2? x<-1或 2? x<3或 x= -1/2 或 x= 0.

12 18

12. 若?x???19??20??21?91??=546,求[100x]的值、 ?x??x????x????????100??100??100??100?y?8,这35项中最小的是等式左100解:等式左侧为73个数相加,而546?73?7?35.?7?x?8,

且可知等式左侧从右向左有且只有35项满足x?侧从右向左第35项x?5757?8,移项得x?7.43. ??100x??743. ,∴ 成立有x?100100习题2-1

1.计算m取何值时,下列各式成立:

(1)32≡11(mod m); (2)1001≡1(mod m); (3)480≡26(mod m); (4)28≡1(mod m). 解:由充要条件知(1)32-11=21,所以m取21,7,3,1. (2)1001-1=1000所以m取1000的所有正约数,共16个; (3)480-26=454=2×227, 所以m取454,227, 2,1.

(4)28-1=(24+1)( 22+1) ( 22-1)= 17×5×3, 所以m取255,85,51,15,17,5,3,1. 2. 计算m取何值时,下列两式同时成立: (1)32≡11(mod m); (2)1000≡-1(mod m) .

一般地,若a≡b (mod m ), c≡d (mod m ) 同时成立,则m要满足什么条件?

解:由m|(32-11)且m|(1000+1)知,m是21和1001的公约数,因此m取7,1. 一般地,若a≡b (mod m ), c≡d (mod m ) 同时成立,则m|(a- b)且m|( c - d ). 3.举例说明:

(1) 由a2 ≡ b2 (mod m ),不能推出 a ≡ b (mod m );

解:(-1) 2 ≡12 (mod5), -1≡1 (mod5)不成立。又如3 2 ≡22 (mod5), 3 ≡ 2 (mod5) 不成立。 (2) 由a ≡ b (mod m ),不能推出a2 ≡ b2 (mod m ). 这是性质(5)的特例,是正确的命题。

4. 证明,?x?Z,( 15x5+24x4+32x3-16x2+3x-13)≡(7x5+3x+3) (mod 8)

证:由15≡7 (mod 8),24≡32≡-16≡0 (mod 8),-13≡3 (mod 8)即可证明上述同余式成立。

5. 证明:70! ≡61! (mod 71)。

证:70!-61!=(70×69×68×?×62-1) 61!

= 23599366843852799×61!= 332385448504969×71×61!

6. 设m为正整数,a,b,c,d为整数,若a ≡b (mod m),c ≡d (mod m) 同时成立,证明 a-c ≡ b-d (mod m).

证:a-c-(b-d)= a-b-(c-d), 而由a ≡b (mod m),c ≡d (mod m),m | (a-b), m |(c-d), ∴ m |[ a-b-(c-d)], 由同余的充分必要条件知a-c ≡ b-d (mod m). 命题成立。 7. (1)求3100模10的余数; (2) 求350的十进制表示中最末两位数。 解:(1)3100≡(32) 50≡(-1) 50 ≡1(mod 10), ∴ 3100模10的余数为1;

(2) 350≡(325) 2 ≡((312) 2×3 ) 2 , 而312≡ ((33) 2) 2 ≡((27) 2) 2 = (729) 2 ≡(29) 2 = 841≡ 41 (mod 100), 412=1681≡81(mod 100) ∴350≡(412×3)2 ≡(81×3) 2≡(243) 2≡43 2=1849≡49 (mod 100), 故所求末两位是49. (现在有计算器,此题可以直接计算350,但数字庞大) 8. 求使2n+1能被5整除的一切正整数n。

解:4≡-1 (mod5),即22≡-1 (mod5), (1) 又16≡1 (mod5), 即24≡1 (mod5), ∴24 k≡1 k=1 (mod5), (2)

(1) (2)两式分别相乘,得22+4k≡-1 (mod5), 即22+4k+1≡5≡0 (mod5),因此n=2+4k, k∈Z+. 说明:4×16=64≡4 (mod5), 64×16≡4×16=64≡4 (mod5), 依此类推可解,以上是抽象推导.

19

9. 设m为正整数,a为整数,若a2 ≡a (mod m),证明 an ≡a (mod m).

证:∵ a2 ≡a (mod m),∴ a3 ≡a2 ∴ a3≡a (mod m),依此类推得,an ≡a (mod m). (最好后面用数学归纳法)

10设p为质数,a为整数,且a2 ≡b2 (mod p) ,证明a ≡b (mod p) ,或a ≡(-b) (mod p) 证:由题设,p | (a2-b) 2, 即p | (a-b) (a+b), p为质数,知p | (a-b), 或p | (a+b),命题成立。 11.用弃九法验算下列算式是否有错:(略)

12.在3145×2653=8□43685中,遗漏了一个数字,如果其他数字都是正确的,求遗漏的数字。

解:3145≡31≡4 (mod 9), 2653≡2+5≡7 (mod 9), 4×7=28≡1 (mod 9), 8□43685≡8+8+□≡7 +□(mod 9), 因7 +3≡1 (mod 9) , 故遗漏的数字为3。

附)广西师范大学赵继源主编的《初等数论》习题2—1中的部分题目 5.若69, 90和125关于某数 d 同余, 证明对于d, 81与 4同余. 证明:由69和90关于 d 同余, d | 90- 69, d | 21,

90和125关于某数 d 同余, d | 125- 90, d | 35, ∴ d | (21, 35) , d=1或7. 2

9. 若(n, 8)=1,试证:n≡1(mod8)

222

证:由 (n, 8)=1可知,n为奇数. 设n=2k+1, n-1= 4k (k+1), 8 | (n-1).所以有n≡1(mod8) 14. 任意平方数的末位数字都不能是 2, 3, 7, 8的某一个.

2 22

证:令a=(10x+y), 则a2=(10x+y)≡y (mod 10). 令=0,1…9, y 的个位不能是2, 3, 7, 8.

2

因此,数字 a (1≤a≤9) 的平方 a的末位数字也没有2, 3, 7, 8.

习题2-2 1,2 验证完全剩余系,简化剩余系。(略)

3. (1)求模9的一个完全剩余系,使其中每个数都是奇数; (2)求模9的一个完全剩余系,使其中每个数都是偶数; (3)对于模10来说,能实现(1)(2)的要求吗? (4)找出规律,证明之。

解:(1)模9的非负最小完全剩余系为:{0,1,2,3,4,5,6,7,8},将其中的偶数减9或加9,如{9,1,11,3,13,5,-3,7,-1};(2)如上,代换其中的奇数,如{0,10,2,12,4,-4,6,-2,8} (3)对于模10来说,不能实现(1)(2)的要求。

(4)对于(1)(2)的要求,奇数模可实现,偶数模不能实现。因为模m的最小非负完全剩余系中奇偶数各半.任一个完全剩余系中各数必与0, 1, …, m-1中的一个数同余,均可写成mkr+r,r= 0,1, …, m-1的形式。而整数加偶数的整数倍奇偶性不变,故偶数模的任一个完全剩余系必奇偶各半,不能实现(1)(2)的要求。奇数模则不然,因为mkr可奇可偶。 或证:若a, b属于模m的同一剩余类,必有a=b+mk,k∈Z, m为偶数时,mk为偶数,故 a, b奇偶性相同,即偶数模的剩余类分为偶数剩余类与奇数剩余类,完全剩余系就是从m的各个剩余类中取一个元素组成的集合。因此不能实现(1)(2)的要求;m为奇数时,若k 取奇数,mk为奇数,k 取偶数,mk为偶数,故a, b奇偶性可不同,即奇数模的任何一个剩余类中都是奇偶混杂,故能实现(1)(2)的要求。

4.将上题中的完全剩余系改成简化剩余系,情况如何?无论奇数模,偶数模均不一定实现。

20

5.设m为正整数,整数集合{x1,x2, …,xm}为模m的一个完全剩余系,则 (1)当m为奇数时,x1+x2+…+xm≡0(mod m); (2)当m为偶数时,x1+x2+…+xm≡

m(mod m) 2证:由3题知,模m的任一个完全剩余系均有xr =mkr+r,r= 0,1, 2,…, m-1。 x1+x2+…+xm≡1+2+…+m-1=

m(m?1)m(m?1),(1)当m为奇数时,≡0(mod m); 22m(m?1)m2?2m?mmmm??m(?1)??(mod m)(2)当m为偶数时,。 222226.设m为大于2的整数,证明:{02,12,22, …, (m-1)2}一定不是模m的一个完全剩余系。

2222

(m-1)-1 = m (m-2), (m-2)-2 = m (m-4) , …,对模m两两不同余。解:由平方差公式,

7. 设m,s,t为正整数,且s >t,证明:整数集合{ x | x= u+mstv,0?u?mst-1 , 0?v?

mt-1}为模ms的一个完全剩余系。

--

证:首先确定集合元素的u有mst 种取法,而u每次取定之后要加不同的mstv,v有mt 种

取法,因此元素总数为mst×mt = ms。然后可以验证题设的集合元素就是0到ms-1的ms个数,它们构成ms的非负最小完全剩余系。证毕。

8. 计算欧拉函数值?(m):?(1236); ?(1218); ?(2001); ?(4200) 解:?(1236)= ?(12×103), 103是质数,因此?(1236)=(22-2)(3-1)(103-1)=408.

?(1218)= ?(6×203) = ?(2×3×7×29)= 2×6×28=336 ?(2001)= (3-1)(23-1) (29-1)=1122 ?(4200)= ?(23×3×52×7)= (23-22)(3-1)( 52-5) (7-1)= 960

9. 设为大于2的整数,证明?(m)为偶数。

证:若m的质因数至少有一奇数,由公式,可知?(m)=

?p?p?1?为偶数。若m的

?i?1iii?1k质因数一个奇数也没有,而m大于2,必有m =2,??1,此时?(m)=2???1,也是偶数。

或证:设 m = p1α1p2α2… pkαk,∵ m >2,∴ 至少存在 i,αi> 1或存在 j,pj是奇数, ∴ p1α1- p1α1 -1,…,pkαk- pkαk-1中至少有一个为偶数,知 φ(m)必为偶数中

又证:若有?i?1,无论pi为奇为偶,有pi?i?pi?i?1=pi?i?1?pi?1?是偶数.所有的?i?1,则必有一个pi为奇数,因为如果m是质数,大于2的质数是奇数,?(m)?m?1是偶数;如果m是合数,pi中最多只有一个是2,那么至少有一个为奇数, 这时?(m)???pi?1?是偶数.

i?1k10. 11(略)

附)广西师范大学赵继源主编的《初等数论》将完全剩余系与简化剩余系分开在2-2,2-3

21

两节, 下面是广西师范大学赵继源主编的《初等数论》习题2-3中的部分题目 1.乘幂 20,21,22,?,29能否构成模 11的一个简化剩余系? 解:i > j时,2-2=2(2

i

j

j

i-j

-1), 11?2, 通过验证可知,对任何i,j,也有11? (2

ji-j

-1),

φ (11) = 10,而20,21,22,?,29为10个不同的整数,所以它们构成模 11的一个简化剩

余系

2.列表求出模 m 为 10,11,12,…,18等值时的最小简化剩余系及相应的φ (m). m 10 11 12 13 14 15 16 17 18 1 1 1 1 1 1 1 1 1 2 2 2 2 3 3 3 3 3 3 4 4 4 4 5 5 5 5 5 5 5 6 6 6 最 小 简 化 剩 余 系 7 7 7 7 7 7 7 7 8 8 8 8 9 9 9 9 9 9 10 11 11 11 11 11 13 13 13 15 17 φ (m) 4 10 4 12 6 8 8 16 6 10 11 12 13 14 10 11 12 13 14 15 16 3.证明定理 2.7:简化剩余系的充分必要条件。 证明:(必要性)∵ x1,x2,…,xk是模 m 的简化剩余系,

∴ k=φ(m),且当 i ≠ j时,xi?,(xi,m)=1,i = 1,2,…,φ(m). ?xj(mod m)(充分性)k=φ(m),∴ x1,x2,…,xk共有φ(m)个. 又 xi?,(i ≠ j,1?i,j? k),(xi,m)=1(i=1,2,…,k), ?xj(mod m)

∴ x1,x2,…,xk各属于φ(m) 个不同的且与 m 互质的剩余类, ∴ x1,x2,…,xk是模 m 的简化剩余系. 4. 验证:(1)8,16,24,32,40,48是模 7的简化剩余系;

(2)11,13,77,99是模 10的简化剩余系. 解:(1)∵(4,7)=1,可化为2,4,6,8,5,12,又5≡12(mod 7),

∴ 8,16,24,32,40,48不是模 7的简化剩余系。

(2)10的最小简化剩余系是1, 3, 7, 9。11,13,77,99分别与1, 3, 7, 9关于模10同余。∴ 11,13,77,99是模 10的简化剩余系. 5. 当 m 取下列各值时,计算φ(m)的值 .

25,32,40,48,60,120,100,200,4200,9450.

答案:φ(25)= 20,φ(32)=16,φ(40)=16,φ(48)= 16,φ(60)=16,φ(120)= 32,

φ(100)= 40,φ(200)= 80,φ(4200)= 960,φ(9450)= 2160.

6. 若φ(m)是奇数,试求 m 的值. 解:(参看下一题) m = 1或 m =2. 7. 当 m >2时,证明φ(m)是偶数 . 8. 试证:使φ(m) =14的数 m 不存在.

证:φ(m) =14=2×7= p1α1 -1…pkαk-1 (p1-1)…(pk-1),2,7是质数,所以必有p1=2,p1=7,这是不可能的。

9. 已知φ(m) = 4,求 m .

解:设m = p1α1p2α2… pkαk,由φ(m)= (p1α1- p1α1 -1)…(pkαk- pkαk-1),φ(m) = 4=4×1=22,得 m = 5,φ(m) =5-1= 4,或 m =8=23,φ(m) = 22或 m = 10=5×2,φ(m) =4×1,或 m =12. 10. 如果 n =2m,(2,m)=1,那么φ(n)= φ(m).

22

解 ∵(2,m)= 1,∴ φ(n)=φ(2m)=φ(2)φ(m)=φ(m). 11. 若 m 是奇数,则φ(4m)=2φ(m).

证:∵ m 是奇数,∴(4,m)= 1,则φ(4m)=φ(4)φ(m).

∵ φ(4)= 2,∴ φ(4m)=2φ(m).

12.(1)分母是正整数 n 的既约真分数的个数是多少?为什么? (2)分母不大于 n 的既约真分数的个数是多少?为什么? 解:(1)φ(n). (2)φ(2)+φ(3)+ ? +φ(n).

习题 2-3

1.举例说明欧拉定理中(a,m)=1是不可缺少的条件 . 解:当 a= 2,m =4时,?(4) =2,此时 22≡0(mod 4),可见(a,m)= 1是欧拉定理的不可缺少的条件.

2. 设 p,q是两个大于 3的质数,求证:p2≡ q2(mod 24). 证:24=3×8,(3,8)= 1. 由条件,( p,3 ) = ( q,3 ) = 1,由费尔马小定理有 p2≡1(mod 3), q2≡1(mod3). ∴ p2 ≡ q2(mod 3).

又 ∵ p,q 必为奇数,设p =2k+1,p2=4k(k +1)+1, 有p2≡1(mod 8), 类似可得q2≡1(mod8).∴ p2 ≡ q2(mod 8). ∴ p2 ≡ q2(mod 24). 3.设 p是大于 5的质数,求证:p4≡1(mod 240).

2

证: 240 = 3×5×16,由条件,(p,3) = (p,5) = 1,∴ p4≡1(mod5),p4≡(p2)≡1(mod3). 又 p为奇质数,从而 2 |(p2+ 1),8 |(p2-1),∴16 |(p4-1),即 p4≡1(mod 16). 而(3,5)=(3,16)=(5,16)= 1. ∴ p4≡1(mod 240). 4. 如果今天是星期一,从今天起再过1010天是星期几?

解:由欧拉定理,106≡1 (mod 7),由普通除法知1010被6除余4, 即1010=6k+4, k∈Z, 故

101010=106k?4??106??64?64,而62≡1 (mod 7), ∴64≡1(mod 7)再过1010天是星期二.

5. 求下列各题的非负最小余数:(1)84965除以13; (2)541347除以17;

(3)477385除以19; (4)7891432除以18; (5)(127156+34)28除以111. 答案:(1)8. (2)10. (3)16. (4)1. (5)70. 解:(1)84965除以13;(13,8)=1, ∴ 812≡1(mod 13),84965=(812)413×89≡1×(-1)4 ×8(mod 13) 或解:82≡-1(mod 13),84965=(82)2482×8 ≡ (-1) 2482 ×8 ≡ 8(mod 13)。

x6. 已知 a =18,m =77,求使 a≡1(mod m)成立的最小自然数 x. 答案:x=30. 1860≡1 (mod 77) ,且满足要求的最小自然数 x必为60 的?(77)??11?1??7?1??60,由定理知,

正约数:1, 2, 3, 4, 5, 6, 10, 12, 15, 20, 30, 由验算可知x=30. 事实上, 由欧拉定理知,

186≡1(mod 7),1810≡1(mod 11),6与10的最小公倍数是30,故有1860≡1 (mod 77) ,而 182≡16 (mod 77) , 183≡57 (mod 77) , 182≡162≡ 25(mod 77) , 185≡57×16≡65 (mod 77) , 186≡(183) 2≡572 (mod 77) , 1810≡185×182≡65×16≡1040≡39 (mod 77) ,(略) 7. 设(m,n)=1,证明:m证:∵(m, n)= 1,∴n对称可得 m

?(n)?(m)10k10?(n)+ n

?(m)≡1 (mod mn).

?(n)≡1(mod m),而 m≡ 0(mod m),∴ m

?(n)?(n)+n

?(m)≡ 1(mod m).

+n

?(m)≡ 1 (mod n). 由同余性质可得 m+n

?(m)≡ 1(mod mn).

23

广西师大本其他题目: 5.已知 p是质数,(a,p)=1,求证:(1)当 a 是奇数时,ap-1+(p-1)a ≡ 0(mod p); (2)当 a 是偶数时,ap-1-(p-1)a ≡ 0(mod p). 证:(1)由 p 是 质 数,(a,p)= 1,a 为 奇 数,有 ap-1≡ 1(mod p).

(p-1)a ≡-1(mod p),∴ ap-1+ (p-1)a≡ 1-1≡ 0(mod p). (2)由条件,ap-1≡1(mod p), (p-1)a≡1(mod p),∴ap-1-(p-1)

?(n)≡1-1≡0(mod p).

6. 已知 p,q 是 两 相 异 的 质 数,且 ap-1≡1(mod q),aq-1≡1(mod p), 求证:apq≡ a(mod pq). 证:∵ ap ≡ a(mod p),∴ apq ≡ (ap)q ≡ aq ≡ a(mod p);同理,apq ≡ (aq)p ≡ ap ≡ a(mod q),

pq

而(p,q)= 1,故 a≡ a(mod pq). 7. 如果(a,m)=1,x≡ ba

?(m)?1(mod m),那么 ax≡ b(mod m).

8. 设 A 是十进制数 44444444的各位数字之和,B 又是 A 的各位数字之和,求 B 的各位

数字之和 .

9. 当 x∈Z 时,求证:(1)2730 | x13- x;(2)24 | x(x+2)(25x2-1). 解答:7. ∵ x≡ba

?(m)?1(mod m),∴ ax≡ aba

?(m)?1≡ a

?(m)b (mod m). ∵ (a,m) = 1,a

?(m)= 1

(mod m),∴ ax≡ b(mod m).

8. 设 B 的各位数字之和为 C,∵ lg44444444= 4444lg4444 < 4444×4= 17776,即44444444 的位数小于17776,∴ A ? 9×17776 = 159984,B < 1 + 9×5 = 46,C ? 4 + 6 = 10. 又 ∵(7,9)= 1,?(9) = 6,4444= 6×740+4,44444444 ≡ 7 4444 ≡ 74 ≡ (-2)4 ≡ 7(mod 9),∴ B 的各位数字之和为 7.

9.(1)∵ 2730=2×3× 5× 7× 13,2,3,5,7,13两两互质,x13- x= x(x12- 1), ∴当 2 | x或 2 | x时都有 x(x12-1)≡ 0(mod 2),x(x12- 1)≡ 0(mod 13). 又 ∵x13-x= x(x6- 1)(x6+ 1),∴ 当 7 | x或 7 | x时都有 x(x6- 1)(x6+ 1)≡ 0(mod 7).而x13- x= x(x4- 1)(x8+ x4+ 1),∴ 当 5 | x或 5 | x时,都有 x(x4-1)(x8+ x4+ 1)≡ 0(mod5).又 x13- x= x(x2-1)(x2+ 1)(x8+ x4+ 1),∴ 当 3 | x或 3 | x时,都有x(x2-1)(x2+ 1)(x8+ x4+ 1)≡ 0(mod3). ∴ 2730 | x13- x. (2)解法一,同上。解法二: x(x+2)(25x2-1)= 24 x3(x+2)+ x(x+2)(x2-1), x(x+2)(x2-1)= x(x-1)(x+1)(x+2),四个连续自然数的乘积必能被4!=24整除,证得。10. 设质数 p>3,x∈Z,试证:6p | xp- x.

?99. 11. p是不等于 2和 5的质数,k是自然数,证明:p|99??????p?1?p个9解答:10. ∵质数 p> 3,∴ (6,p)=1,xp- x= x(xp-1- 1)≡ 0(mod p). 又 p- 1是偶数,

∴x(xp-1-1)≡ x(x2- 1)?(mod p). 于是,当 2 | x或 2 | x 时,x(x2- 1)≡ 0(mod 2);当 3 | x或 3 | x时,x(x2-1)≡ 0(mod 3).故 x(xp-1- 1)≡ 0(mod 6).从而6 | p(xp- x).

?99?1011. 99??????p?1?k个9?p?1?k?1. 由条件,(10,p)= 1,∴ 10p-1≡ 1(mod p).

?99. ∴ (10p-1)k≡ 1(mod p). ∴ p|99??????p?1?p个9(王进明与赵继源本三、四章编排顺序相反) 习题3-1(P157)

24

1.试写出三个模数是20的一次同余方程,分别使它有唯一解,无解,有四个解。 解:7x≡50(mod 20)有唯一解,8x+5≡0(mod 20)无解, 4 x≡0(mod 20)有四个解。 2. 下列同余方程是否有解?为什么?如果有解,不出其所有解。

(1)7x+4≡0(mod 25);(2)12x+1≡0(mod 9);(3)34x≡1(mod 51); (4)42x≡8(mod 138);(5)174x≡65(mod 1309).

解:(1)(7, 25)=1, 有唯一解。7x≡-4(mod 25)由欧拉定理,?(25)=20,故x≡719×(-4) ≡3 (mod 25),化简过程:72≡-1(mod 25)故719×(-4) ≡718×7 (-4) ≡(-1)×7 (-4) ≡28≡3 (mod 25). 用同余变形法解:x??4?25?3. 7(2)(12, 9)=3?1, 同余方程无解;(3)(34,51)=17?1, 同余方程无解;

(4)138=2×3×23,(42,138)=6不能整除8, 同余方程无解;

(5)174=2×3×29, 1309=7×11×17,( 174, 1309)=1.此题数字大,应该用大衍求一术。

1740130907121817419191183108023183 862n q P 1 0 1 2 7 7 3 1 8 4 1 15 5 10 158 6 2 331 7 1 489 x≡489×65=31785≡369(mod 1309) 3.求下列一次同余方程的所有解:(1)3x≡1(mod 17);(2)1215x≡560(mod 2755); (3)47x≡89(mod 111);(4)38x≡6(mod 106);(5)11x≡6(mod 13). (6)3x≡5(mod 29);(7)5x≡6(mod 24)(8)66x≡14(mod 74); 解:(1)(3, 17)=1, 有唯一解。由3x≡1+17(mod 17)得,x≡6(mod 17);

(2)(1215,2755)=5(243,551)= 5(35, 19×29), 方程恰有5解,先解243x≡112(mod 551): 这个系数相当大,故应该用大衍求一术。辗转相除过程的竖式略,由如下表格

n q P 1 0 1 2 2 2 3 3 7 4 1 9 5 2 25 6 1 34 7 4 161 8 1 195 9 1 356

知x≡112×356≡200(mod 551), 故原方程有解x≡200751, 1302,1853和2404(mod 2755). (3)(47,111)=1,方程恰有一解x≡89(mod 111);由欧拉定理,?(111)=72,故x≡4771×(89) ≡94(mod 111),用同余变形法解:x?89?11120025136???????17(mod 111)

47?111?6488(4)38x≡6(mod 106);先解19x≡3(mod 53). 用同余变形法解有

25

x?3?5356282?53???????11?42(mod 53)

19?53?3417?53511?1原同余方程的解有两个:x≡42(mod 106),x≡95(mod 106) (5)11x≡6(mod 13).x???1??6?12?11?10???4?3?6?6?10(mod 13)

11!(6)3x≡5-29=-24(mod 29), ∴ x≡-8=21(mod 29); (7)5x≡6+24=30(mod 24), ∴ x≡6(mod 24) (8)66x≡14(mod 74);先解33x≡7(mod 37);

x???1?33?1?7?36?35?34???6?536?35?34?7??7???6??17≡26(mod 37)

33!4?3?2原同余方程的解有两个:x≡26(mod 74), x≡63(mod 74)。 下面是广西师大本中要求按指定方法求解的题目,可自行练习:

4.化为不定方程解下列同余方程(王进明本第四章讲不定方程,此题建议用同余变形法): (1)20x≡4(mod 30);(2)57x≡87(mod 105); (3)4x≡11(mod 15); 5.利用欧拉定理解下列同余方程:

(1)6x≡22(mod 36); (2)3x≡10(mod 29); (3)258x≡131(mod 348); (4)11x≡7(mod 13); (5)3x≡2(mod 17); (6)243x≡102(mod 551). 6.用求组合数的方法解下列同余方程:

(1)5x≡13(mod 43); (2)9x≡4(mod 2401); 王进明本习题3.2(P169) 赵继源本习题4-3(P154)

解略

2. a取什么值时,下面的同余方程组有解?

?x?5(mod18),?x?5(mod18),解:因为(18, 21)=3, 3|8-5,所以?有解;(18, 35)=1, 所以?x?8(mod21).??x?a(mod35).总是有解。因此,要使题设的同余方程组有解,只需??x?a(mod35),有解。

?x?8(mod21).而这里,(21, 35)=7,由定理可知,只需a≡8≡1(mod 7). 即x = 1+7t ( t为整数 ),题设的同余方程组总有解。

26

3.解下列同余方程组:(1) ??x?3(mod7),, (2)

?x?5(mod11).?x?6(mod13), ??x?7(mod24).?2x?4(mod8), ??15x?5(mod35).(4) ??5x?7(mod11), (5)

?6x?9?0(mod19).解:(1)方法一:设x=5+11y, 代入第一个同余方程,得11y≡3-5 (mod 7), 得y≡3 (mod 7) 所以同余方程组的解是x≡38 (mod 77)。 方法二:用孙子定理解,M=77,M1=11,M2= 7, 令11 M1′≡1(mod 7), 得M1′≡2(mod 7), 7 M2′≡1(mod 11),得M2′≡-3(mod 11),

所以同余方程组的解是x≡11×2×3 + 7×(-3)×5≡-39≡38 (mod 77). (2) 方法一:设x=7+24y, 代入第一个同余方程,得24y≡6-7 (mod 13), 得y≡7 (mod 13) 所以同余方程组的解是x≡7+24×7≡175 (mod 312)。 方法二:用孙子定理解,M=312,M1=24,M2= 13, 令24 M1′≡1(mod 13), 得M1′≡6≡-7 (mod 13), 13 M2′≡1(mod 24),得M2′≡13(mod 24),

所以同余方程组的解是x≡24×(-7)×6 + 13×13×7≡-1008+1183≡175 (mod 312). 4. 求解下列各题(我国古代数学家杨辉 1275 年所写《续古摘奇算法》中的三个例题): (1)七数剩一,八数剩一,九数剩三,问本数;

(2)十一数余三,十二数余二,十三数余一,问本数; (王进明攺了几个数) (3)二数余一,五数余二,七数余三,九数余四,问本数.

??x ? 1 解:(1)???x ?3?mod 56?,方法一:设x=1+56y, 代入第二个同余方程,得

?mod 9?.56y ≡ 3-1 (mod 13),(56-54)y ≡ 2 (mod 13)

得y ≡ 1 (mod 13) y,所以,同余方程组的解是x≡57 (mod 504)。 方法二:用孙子定理解。

?x ? 3 ?(2)?x ? 2 ??x ?1 ?,?mod11 ?, ?mod12 ?.?mod13用孙子定理解,M=1716,M1=156,M2=143,M3=132,

令156 M1′≡1(mod 11), 得2M1′≡12(mod 11),M1′≡6(mod 11);

143 M2′≡1(mod 12), 得-M2′≡1(mod 12),M2′≡-1(mod 12); 132 M3′≡1(mod 13), 得2M2′≡-12(mod 13),M2′≡-6(mod 13)

所以同余方程组的解是x≡156×6×3 + 143× (-1)×2 +132×(-6)×1 ≡ 14(mod 17). 法二:观察法。或累加试除法。

5. 求相邻的四个整数,它们2,3,5,7 依次可被整除。

2222?4x ? 1 ?0?mod 32?,?4x ??1?mod 32?,?x ?2?mod 9?,?????22解:由题意得?4x ?2? 0 ?mod 5?,即?4x? ?2 ?mod 5?,即?x? ?13 ?mod 25?,

???224x?3 ?0mod 7.4x ??3mod 7.??????x ??13?mod 49?.??? 27

??x ?2?mod 9?,用孙子定理解,M=11025,M1=1225,M2=9, ???x? ?13 ?mod 25?49?.令1225M1′≡1(mod 9), 得M1′=1; 9M2′≡1(mod 1225), 1225=9×136+1,∴ 9×(-136)+1225=1,即M2′=-136 所以x≡1225×1×2 + 9× (-136)×(-13) =18362 ≡ 7337 (mod 11025). 所求的四个连续整数为29348,29349,29350,29351.

6. 求模11的一个完全剩余系,使其中每个数被2,3,5,7除后的余数分别为1,-1,1,-1。

?x ?1?mod 2?,? ?,??x ??1?mod 3?,?x ?1?mod10解:先求x, 满足:?等价于?

??x? 1 ?mod 5?,?x ??1?mod 21?.?x ??1mod 7.??? 41?mo d?210,???x ?41?mod 210?,?x?可解得x ?41?mod 210?.再依次解? ?

?. d?11.???x?0?mod11?x?1?mo??x ?41?mod 210?,??(略) ?x?2mod11 .????以下为赵本的第4题:

5..解下列同余方程组: (1) x≡8(mod 15), (2) x≡6(mod 11),

x≡3(mod 10), x≡3(mod 8), x≡1(mod 8); x≡11(mod 20); (3) x≡2(mod 35), (4) 4x≡90(mod 105),

x≡9(mod 14), 5x≡18(mod 63), x≡7(mod 20); 7x≡10(mod 50),

3x≡12(mod 22)

?x ? 2 ?解:(1)化为?x ? 3 ??x ?1?mod 3?,?mod 5?,用孙子定理解,M=120,M1=40,M2=24,M3=15, ?mod 8?.令40 M1′≡1(mod 3), 得M1′≡1 (mod 3);

24 M2′≡1(mod 5), 得-M2′≡1(mod 5),M2′≡-1(mod 5); 15 M3′≡1(mod 8), 得-M2′≡1(mod 8),M2′≡-1(mod 8)

28

所以同余方程组的解是x≡40×2 + 24× (-1)×3 +15×(-1)×1 ≡ -7(mod 120).

?x ? 2 ?(3)化为?x ? 9 ??x ?3?mod 35?, ?,?mod1?mod 4?.??x ? 2 即???x ?3?mod 35?,

?mod 4?.用孙子定理解,M=140,M1=4,M2=35, 令4 M1′≡1(mod 35), 得M1′≡9 (mod 35);

35 M2′≡1(mod 4), 得-M2′≡1(mod 4),M2′≡-1(mod 4);

所以同余方程组的解是x≡4×9×2 + 35× (-1)×3 ≡ -33 ≡ 107 (mod 140).

7. 设韩信所辖某部士兵共 26641人,在一次战斗中损失近百人. 休整时清查:1~3报数余 1,1~5报数余 3,1~ 7报数余 4. 问损失了多少人?

?x ? 1 ?mod 3?,??x ? 3 ?mod 5?,解:设还有士兵x人,由题设,得同余方程组?

?x ?4?mod 7?,?26541?x ?26600.?由口诀,x≡70 + 21×3 + 15×4≡193+105×251≡ 26548 (mod 105) 26641-26548=93人。损失了93人。

?26641?x ? 1 ??26641?x ? 3 设损失了x人,得同余方程组??26641?x ?4?0?x ?100.??x ? 0 ?mod 3?,?mod 3?,??mod 5?,?x ? 3 ?mod 5?,化为?

?mod 7?,?x ?2?mod 7?,?0?x ?100.?由口诀,x=21×3 + 15×2=93。损失了93人。

8. 求 7的倍数,使它分别被 2,3,4,5,6除时,余数都是 1.

解:设所求为7x, [2,3,4,5,6] =60,由题设,得7x ? 1 ?mod 60?,

用大衍求一术得x≡43 (mod 60), 7x=7×43=301, 故,所求为301+420t , t 为整数。 9. 求三个连续的自然数,使它们从小到大依次被 15,17,19 整除(写出其中最小的一组).

??15x?1 ? 0 解:由题设,得同余方程组???15x ?2?0 ?, ?,??mod17?x ? 9 ?mod17化为标准形式为?

mod19 .x ?10mod19 .??????用孙子定理解,M=321,M1=19,M2=17,

令19 M1′≡1(mod 17), 得M1′≡9(mod 17), 17 M2′≡1(mod 19), 得M2′≡9(mod 19),

所以同余方程组的解是x≡19×(-8)×9 + 17×9×10 ≡ 162 (mod 321). 本题所求的三个连续的自然数是162×15,162×15+1,162×15+2,即2430,2431,2432。

下面是不定方程内容(王进明本为第四章,习题这里没做)(赵继源本)习题3-1 1.解下列不定方程:

(1)7x-15y=31; (2)11x+15y=7; (3)17x+40y=280;

29

(4)525x+231y=42; (5)764x+631y=527; (6)133x-105y=217. 解:(1)辗转相除得15=7×2+1, ∴ 1 = 15-7×2= 7×(-2)-15×(-1), ∴ 因此原方程的一个解是 x0=-2×31=-62, y0=-1×31=-31;

?x??62?15t原方程的通解为?这里t为任意常数.

?y??31?7t(2)辗转相除得15=11×1+4, 11=4×2+3, 4=3+1 ∴ 1 = 4-3=4-(11-4×2)= 4×3-11=

(15-11×1) ×3-11=15×3 + 11×(-4),

∴ 因此原方程的一个解是 x0=-4×7=-28, y0=3×7=21;

?x??28?15t原方程的通解为?这里t为任意常数.

?y?21?11t(3)用分离整数法:x?280?40y8?6y?16?2y?.

1717观察可知y =-10时,x = 36 + 4= 40.

?x?40?40t∴ 原方程的通解为?这里t为任意常数.

y??10?17t?2. 解下列不定方程:(1)8x-18y+10z=16; (2)4x-9y+5z=8; (3)39x-24y+9z=78; (4)4x+10y+14z+6t=20; (5)7x-5y+4z-3t=51.

3. 解下列不定方程组:(1) x+2y+3z=10, (2) 5x+7y+3z=25,

x-2y+5z=4; 3x- y-6z=2;

(3) 4x-10y+ z=6, (4) 10x+7y+ z=84,

x-4y- z=5; x-14y+ z= -60;

4. 求下列不定方程的正整数解:(1)5x-14y=11; (2)4x+7y=41; (3)3x+2y+8z=21. 5. 21世纪有这样的年份,这个年份减去 22 等于它各个数字和的495倍,求这年份.

6. 设大物三值七,中物三值五,小物三值二,共物一百三十八,共值一百三十八,问物大中小各几何?

7. 买2元6角钱的东西,要用1元、5角、2角、1角的四种钱币去付,若每种钱币都得用,则共有多少种付法?

8. 把 239分成两个正整数之和,一个数必是 17 的倍数,另一个数必是 24的倍数,求这两位数.

9. 一个两位数,各位数字和的 5倍比原来大 10,求这个两位数.

10. 某人 1981年时的年龄恰好等于他出生那一年的年号的各位数字之和,这个人是在哪一年出生的?

11. 一个四位数,它的个位数上数比十位数字多 2,且此数与将其数字首尾颠倒过来所得的四位数之和为 11770,求此四位数 . 习题 3-2

1.求 x2+ y2= z2中 0< z<60的所有互质的解.

2.求三个整数 x,y,z(x> y> z>0),使 x- y,y- z,x- z都是平方数 . 1.

b 1 1 1 2 2 2 3 4 30

本文来源:https://www.bwwdw.com/article/n556.html

Top